John bought a motor cycle for $56,750 and spent $4530 On its repair. Then he sold it for $72650, what was the profit?​

Answers

Answer 1

Answer:

cost of old motor bike =$5675

he spent on repair =$453

cost that he sold =$7265

profit he made=$7265-$5675+$453

=$7265-$6128

=$1137

Step-by-step explanation:


Related Questions

PLEASE HURRY

Let p: A student plays basketball.

Let q: A student plays tennis.




How many students play both basketball and tennis?

Answers

Answer:

4

Step-by-step explanation:

4 is the intersecting point which states p intersect q giving both tennis and basketball players

Help Me Uhm In 5th Grade I'm behind in school work and I have to finish by august when my school starts pls help I'm stuck on this question

Answers

B

To find the average you add them all up and divide by how many numbers there are. 2+3+6+13+16=40 There are 5 numbers so 40/5=8

A would be 7 and c would be 9, so the answer is b.

Prove the triangles a re similar

Answers

Step-by-step explanation:

LINE B.I is parallel to LINE O.D

therefore :Ô = Î ( alternate angles )

: Ď = B ( alternate angles )

In ️ B. I. G and ️ D O G

O = I ( alternate angles)D = B ( alternate angles)G = G ( 3rd angle of️ )

️ BIG is similar to ️ DOG


(a - b)(2x-3)+(6x+7)(a+b)
Factorise

Answers

Answer:

2(4ax+2bx+2a+5b)

Step-by-step explanation:

How many calories does the bran muffin contain? 400 calories 200 calories 650 calories 800 calories

Answers

Answer:

400 calories

Step-by-step explanation:

A bran muffin has around 380 calories, when rounded off it is 400 calories.

Hope I helped

Answer:

A. 400 Calories.

Step-by-step explanation:

Look it up on the net.

If this is wrong, it is because there wasn't enough information given.

Hope this helps!

If not, I am sorry.

(x - 4)/(x + 6) - (x - 6)/(x + 4)​

Answers

Answer:

[tex]\frac{20}{(x+4)(x+6)}[/tex]

Step-by-step explanation:

assuming you require the expression simplified

[tex]\frac{x-4}{x+6}[/tex] - [tex]\frac{x-6}{x+4}[/tex]

multiply the numerator/denominator of the first fraction by (x + 4)

multiply the numerator/denominator of the second fraction by (x + 6)

= [tex]\frac{(x-4)(x+4)}{(x+4)(x+6)}[/tex] - [tex]\frac{(x-6)(x+6)}{(x+4)(x+6)}[/tex] ← expand and simplify numerators leaving the common denominator

= [tex]\frac{x^2-16-(x^2-36)}{(x+4)(x+6)}[/tex]

= [tex]\frac{x^2-16-x^2+36}{(x+4)(x+6)}[/tex]

= [tex]\frac{20}{(x+4)(x+6)}[/tex]

Select the true statement about triangle ABC.
13
5
12
A. sin C = sin A
B. sin C = cos B
C. sin C = tan A
D. sin C = cos A

Answers

Answer:

sin C = cos A

Step-by-step explanation:

sinC = opp/hyp

=12/13

cosA = adj/hyp

12/13

A man of height 1.5 m is standing in front of a tower of height 81.5 m. At what angle should he observe the top of the tower from the distance of 80 m?​

Answers

The angle at which the man should observe is 45 degrees.

To solve the question :

Given :

height of man = 1.5m

height of tower = 81.5 m

distance between tower and man = 80 m

Since, ABCD is a rectangle, AD = BC = 80m

Similarly, AB = CD = 1.5m

EC = ED + CD

81.5 = ED + 1.5

ED = 80m

Now, in order to find the angle EAD, we use trigonometric functions,

tan ∅ = perpendicular / base = ED / AD

tan ∅ = 80 /80 = 1

∅ = [tex]tan^{-1} (1)[/tex]

∅ = 45°.

Therefore, The angle at which the man should observe is 45 °.

To learn more about height of a tower refer to : https://brainly.com/question/16136347

#SPJ2

ax +by = 16
3x+2y=64

In the system of equations, a and b are constants. if the system has infinitely many solutions, what's the value of ab?

Answers

Answer:

The system has infinitely many solutions when a = 3/4 and b = 1/2

Given

System of linear equations

ax +by = 163x+2y=64

To find

Values of a and b that leads to infinitely many solutions

Solution

The linear system of equations has infinitely many solutions when lines overlap, so both have same slope and y-intercept.

We can solve it in two different ways

1. Note that when the first equation is multiplied by 4, it has same value of constant as the second equation, which is 64.

4ax + 4by = 643x + 2y = 64

When compared it gives us:

4a = 3 ⇒ a = 3/44b = 2 ⇒ b = 2/4 = 1/2

2. Convert both equations from standard to slope-intercept form and compare:

ax + by = 16by = -ax + 16y = - (a/b)x + 16/b

3x + 2y = 642y = - 3x + 64y = - (3/2)x + 32

Work out values of a and b:

16/b = 32 ⇒ b = 16/32 ⇒ b = 1/2- (a/b) = - 3/2 ⇒ a/b = 3/2 ⇒ a = 3b/2 ⇒ a = 3(1/2)/2 ⇒ a = 3/4

y = 2x² - 9x + 10
solve the following quadratic function by factoring

Answers

hey there! ;)

y = (x - 2)(2x - 5)

please give it a like :)

WILL GIVE BRAINLYEST
Find measure of X and Y
X=

Y=

Part B:
Choose the correct degree measure for JKL
A. 98
B. 65.5
C. 262
D. 24.5

Answers

Part a
X=14
Y=8
Part b
C

Answer:

x = 7 y = 4 JKL = 262 degrees

Step-by-step explanation:

According to a circle theorem, sum of the opposite angles in a circle which touch its circumference must equal 180, hence:

7x + 131 = 180

x = 7

According to another theorem, a quadrilateral inside a circle must equal 360 so:

99 + 7(7) + 131 + 11(7) + y = 360

Note: remember to substitute 7 for all values of x

y = 4

The angle subtended by an arc at the centre is twice the angle subtended at the circumference

Following this rule, we can conclude that:

2(7x) or 2(2 x 7) = 2(49) which is 98

Lastly, 360 - 98 = 262

Hii! would anyone mind helping me out?

Find the center of this hyperbola

[tex]\large\boldsymbol{9x^2-y^2-72x+8y+119=0}[/tex]

[tex]\bigstar[/tex]

[tex]\underbrace[/tex]

Answers

Answer:

(4, 4)

Step-by-step explanation:

This is the form of a hyperbola. Use this form to determine the values used to find vertices and asymptotes of the hyperbola.

[tex]\frac{(x-h)^2}{a^2} - \frac{(y-k)^2}{b^2} = 1[/tex]

Match the values in this hyperbola to those of the standard form. The variable h represents the x-offset from the origin, k represents the y-offset from origin, a.

a = 1

b = 3

k = 4

h = 4

Thus,

[tex](x-4)^2 - \frac{(y-4)^2}{9} = 1[/tex]

The center of a hyperbola follows the form of (h, k). Substitute in the values of h and k.

= (4, 4)

Here is a rectangle.
A (-3,0)
D(-2,-3)
B (3,2)
C (4,-1) What is CD unit number?

Answers

Step-by-step explanation:

Let the area of rectangle with given points are as follows :

A(-3,3)

B(3,2)

C(4,-1) = C( x1,y1)

D(-2,-3) = D( x2, y2)

Now,

The distance of CD = root under

(x2-x)^2 +( y2 - y1)^2

= root under (-2-4)^2 + ( -3+1)^2

= root under (-6)^2 + (-2)^2

= root under 36 + 4

= root under 40

= 2root 10

Therefore; the unit number if CD is 2 root 10.

WILL GIVE BRAINLIEST!!!
For science class, Cora observed the growth of a pepper plant over time. Below is her log. How much taller was the pepper plant at the end of 4 weeks?

Growth of Plant
Start 3 inches
Week 1 5 inches
Week 2 7 inches
Week 3 9.5 inches
Week 4 11 inches

A. 6.75 inches

B. 7.5 inches

C. 8 inches

D. 10 inches

Answers

Answer:

c

Step-by-step explanation:

week 4 = 11

start = 3

11 - 3 = 8

answer is c

The correct Answer is: C. 8 inches

Step-by-step explanation:

If the plant started out at 3 inches and finished after 4 weeks at 11 inches, we can do 11-3 to find how much growth the pepper plant had.

11 - 3 = 8, therefore our answer is

C. 8 inches

The men took an average of 21.89 hours of personal time per year with a standard deviation of 3.29 hours. The Human Resources Department believes that women tend to take more personal time than men because they tend to be the primary child care givers in the family. The t-test for two means is appropriate in this situation because Group of answer choices women and men are dependent samples. women and men are independent samples. women and men are matched samples. the observations are paired. None of these.

Answers

The t-test for the two means is appropriate in this situation because women and men are independent samples.

What are Independent Samples?

Independent samples are chosen randomly, ensuring that their results are independent of other observations' values.

The premise that samples are independent underlies many statistical analyses. Others are made to evaluate non-independent sample sets.

The samples of men and women are independent since their results are independent of other observations' values.

The Independent Samples t-Test analyzes the means of two separate groups to see if there is statistical support that the population means values are statistically substantially different.

Learn more about Independent Samples here:

brainly.com/question/14099859

#SPJ4

Desiree is given an aptitude test with 50 multiple-choice questions. For every correct answer, Desiree will get 3 points. For every wrong answer, 1 point will be deducted. For every question unanswered, 0.5 point is deducted. Desiree did not leave any question unanswered and gets 110 points on the test.
If x is the number of questions Desiree answered correctly, then the equation that represents the given situation is
and the equation will have
.

Answers

Total number of multiple-choice questions that Desiree has to answer in Aptitude test = 50

Points given for every correct Answer = +3

Points deducted for every Incorrect Answer = -1

For every question unanswered ,

points Deducted = -0.5

Total Points Obtained by Desiree after Answering all the questions = 110

Number of Answers that Desiree answered correctly = x questions

Number of Incorrect Answers = (50 - x) questions

Then,the Equation representing above situation

→ 3 × x + ( -1 ) × ( 50 - x ) = 110

⇒3x - 50 + x = 110 ----------- equation that represents the given situation

⇒ 4x - 50 = 110

Adding 50, on both sides

→ 4x - 50 + 50 = 110 + 50

⇒ 4x = 160

Dividing both sides by, 4 we get

x = 40

Number of correct answers given by Desiree= 40 questions

Number of Incorrect Answers = 50 - 40

= 10 Questions .

Answer:

Desiree is given an aptitude test with 50 multiple-choice questions. For every correct answer, Desiree will get 3 points. For every wrong answer, 1 point will be deducted. For every question unanswered, 0.5 points is deducted. Desiree did not leave any questions unanswered and gets 110 points on the test.

If x is the number of questions Desiree answered correctly, then the equation that represents the given situation is

3(50 - x) + x = 110

and the equation will have

no solution

.

Step-by-step explanation:

dsfghjmbhjghcvjkjjbhvgjjbkjguy8uygiyugykfgjhgghgjk

Answers

The equation y = 1/3x - 2 passes through the diagonals of ZDRN

How to determine the end result?

The points are given as:

Z = (-3, -1)

D= (3, -1)

R = (3, -3)

N = (-3, -3)

The equation of the line is given as:

y = 1/3x - 2

When the points and the line equation are plotted in a graph on the same coordinate (see attachment), we can see that the equation y = 1/3x - 2 passes through the points Z = (-3, -1) and N = (-3, -3)

These points are the diagonals of the shape formed by the four points

Hence, the equation y = 1/3x - 2 passes through the diagonals of ZDRN

Read more about linear equations at:

https://brainly.com/question/14323743

#SPJ1

A company ordered $6,000 worth of chairs. Some of the chairs ordered cost $20 each and the others cost $40 each. If twice as many $20 chairs as $40 chairs were ordered, how many chairs were ordered altogether

Answers

In total 180 chairs are ordered for the cost $6000.

The total cost of chairs=$6,000.

What are simultaneous equations?

In mathematics, a set of simultaneous equations, also known as a system of equations or an equation system, is a finite set of equations for which common solutions are sought.

Let x be the chairs ordered each at $20 and y be the chairs ordered each at $40.

Then equation will be 20x+40y=6000-------(1)

Given that, twice as many $20 chairs as $40 chairs were ordered.

That is 2x=y⇒2x-y=0--------(2)

Multiply equation (2) by 40.

That is 80x-40y=0--------(3)

By adding equation (2) and (3) we get

20x+40y+80x-40y=6000

⇒100x=6000

⇒x=60

Substitute x=60 in equation (2)

That is, 2x=y

⇒y=120

Total number of chairs=x+y=60+120=180 chairs.

Hence, in total 180 chairs are ordered for the cost $6000.

To learn more about simultaneous equations visit:

https://brainly.com/question/16763389.

#SPJ1

Javier is comparing costs of two cars. Car A costs $130 per month for insurance and $0.10 per mile to run. Car B costs $108 per month for insurance and $0.15 per mile to run. He defines x as the number of miles driven each month and y as the total cost to run the car. Which set of equations can be used to find the number of miles Javier would need to drive to make the costs equal?
0.10 x = 130. 0.15 y = 108
y = 0.10 x + 130. y = 0.15 x + 108
0.10 x = 108. 0.15 y = 130
y = 0.15 x + 130. y = 0.10 x + 108

Answers

Answer:

y = 0.10 x + 130. y = 0.15 x + 108

Step-by-step explanation:

0.10 and 0.15 are the slope in the equation y = mx + b since it requires an input (number of miles) to spit out an output (number of miles*0.10 = price) and the x intercept is 130 and 108 creating the equation  y = 0.10 x + 130. y = 0.15 x + 108.

WILL MARK BRAINLEST
Fill in the missing work and justification for step 5 when solving 4(x + 1) = 8.



Step Work Justification

1 4(x + 1) = 8 Given

2 4x + 4 = 8 Distributive Property

3 4x + 4 − 4 = 8 − 4 Subtraction Property of Equality

4 4x = 4 Simplify

5

6 x = 1 Simplify

A 4x − 4 = 4 − 4; Subtraction Property of Equality

B 4x + 4 = 4 + 4; Addition Property of Equality

c Four times x over four equals four over four; Division Property of Equality

d 4x(4) = 4(4); Multiplication Property of Equality

Answers

Answer:

c) Four times x over four equals four over four; Division Property of Equality

===========================

Given equation and steps to solve it:

4(x + 1) = 84x + 4 = 84x + 4 - 4 = 8 - 44x = 44x/4 = 4/4                        This is the missing stepx = 1

All steps given above including the missing one. As we can see, we divided both sides by 4, so the justification for this step is Division Property of Equality.

Hence, the correct choice is C.

Answer:

Four times x over four equals four over four; Division Property of Equality

Option C

Step-by-step explanation:

See on antepenultimate step

4x=4

Now for getting x we have to divide each side by 4

We get.

4x)4=4/4x=1

lcm of 15,18,21
lcm of 56 84 140
lcm

Answers

Step-by-step explanation:

The lcm of first question is 630 and second is 840

Answer:

LCM of 15,18,21: 630

LCM of 56,84,140: 840

Step-by-step explanation:

The right method is to list all the multiples of the number and circle the least common ones. In this way you will get your answer for any LCM question!

What is the constant in the expression?

Answers

Answer:

12

Step-by-step explanation:

A constant in an expression is the number that is not attached to any variable, and this includes the sign that comes with the number.

0.05m +12

In the expression above, m is the variable thus 0.05 is known as the coefficient of m instead of a constant. Coefficients are numbers that are multiplied to variables.

+12, which can simply be written as 12, is not multiplied to any variable and is thus the constant.

The value of the number 12 will not change no matter the value of m. The fact that the value stays the same makes it a constant.

Additional:

To learn more about constant, check out the following!

https://brainly.com/question/16517603

Answer: 12 ✅

Step-by-step explanation:

Hii, I'd love to help you! (:

This problem is about finding the constant in the expression. This is fun! Let's start our search...

____________________________________________

Wait, how can we find a constant if we don't even know what it is?

No reason to panic!

Constants

A constant is a number that is not added (or subtracted) from any variables (and a variable is a letter).

_____________________________________________

Ohh now it's easy as winking!

Because the only number in this particular expression that is not added to any letters at all is 12.

______________________________________________

Hope I helped! Best wishes.

Reach far. Aim high. Dream big.

[tex]\underbrace[/tex]

Pleaseeeeee helpppppp

Answers

They are both factors of 15x-10 because 5(3x-2)=15x-10

factor x factor=product.

In this case 15x-10 is the product

Carly will be going to college in 3 years. she anticipates that she will need $12,000 to pay for the first year. she currently has $2,900 in a savings account. without including any interest earned, what is a reasonable estimate of the amount carly needs to deposit into the savings account per month over the next 3 years to be able to pay for her first year of college? $150 $250 $350 $450

Answers

The estimate of the amount carly needs to deposit into the savings account per month over the next 3 years to be able to pay for her first year of college is $250

Savings

Amount needed = $12,000Amount available = $2,900Number of months in three years = 12 × 3 = 36Amount of savings per week = x

12,000 = 2900 + 36x

12,000 - 2900 = 36x

9,100 = 36x

x = 9100 / 36

x = 252.777777777777

Approximately,

$250

Learn more about savings:

https://brainly.com/question/25787382

#SPJ1

Answer:

The answer is B. $250

Step-by-step explanation:

? Question The point (x, y) is first rotated 180° clockwise about the origin, translated 6 units to the left, and then reflected across the line y = x. Write a function S to represent the sequence of transformations applied to the point (x, y).

Answers

The function S to represent the sequence of transformations applied to the point (x, y) is  (-y, -x - 6)

Transformation of coordinates

Transformation is a technique used to change the position of an object on an xy-plane.

Given the coordinate (x, y)

Rotation of the coordinate by 180° clockwise about the origin will given;

(x, y) - > (-x, -y)

If the result is translated 6 units to the left, then;

(-x, -y) -> (-x-6, -y)

If it is then reflected across the line y = x, we will switch the coordinates to have:

(-x-6, -y) -> (-y, -x - 6)

The function S to represent the sequence of transformations applied to the point (x, y) is  (-y, -x - 6)

Learn more on transformation here: https://brainly.com/question/1548871

#SPJ1

A student is trying to solve the system of two equations given below:
Equation P: x+y=8
Equation Q: 2x + 5y = 24
Which of the following steps can be used to eliminate the x term?
02(x+y=8)
O-2(x+y=8)
O-1(2x + 5y = 24)
O-2(2x + 5y = 24)

Answers

Step-by-step explanation:

as the following pic answer should be × -2

Answer:

O -2(x+y=8)

Step-by-step explanation:

If you put the problems beside each other, they look like this;

2x + 5y = 24

  x + y = 8

So you are trying to find a way to simplify the x's

Step 1;

See if there are any numbers that you could multiply to the bottom problem that would help you get rid of the x specifically.

In this case, -2 would cancel out that other 2 right? Because if you added them, they cancel each other out; 2 -2 = 0

Step 2:

if you multiply, you have to do it to the entire problem;

-2(x + y = 8)    (-2 times x, -2 times y, -2 times 8)

-2x + -2y = -16

Step 3:

Go back to your original problem and work it out;

2x + 5y = 24

-2x + -2y = -16

2x and -2x cancel each other out so now you only have;

5y = 24

-2y = -16

Step 4:

Combine like terms;

5y = 24

-2y = -16

3y = 8

Divide by 3

y = 2.6

HELP I REALLY NEED HELP

Answers

Answer: 72 inches squared

Step-by-step explanation:

So, the real floor of a classroom is 36 feet by 32 feet.

And the scale drawing, has length of the classroom equal to 9 inches.

What is the area in square inches of the floor in the scale drawing

1 foot = 12 inches

The length of the classroom, measured in feet, has been multiplied by 12, to get converted in inches. After that, it has been divided by a number, a proportion, to get to 9 inches.

36feet * 12 / x = 9 inches

432 inches / x = 9 inches

Now, to solve for x, we can cross-multiply

432 inches = 9x inches

Divide 9 inches on both sides.

x = 48

To solve the exercise, we need to calculate the floor's area in the scale drawing.

We have the length = 9 inches.

Now, let's calculate the width

Width = 32 feet * 12 / 48

Width = 384 inches / 48

Width = 8 inches

Area = Length * Width

Area = 9 inches * 8 inches

Area = 72 inches squared

Hope I helped!

What can each term of the equation be multiplied by to eliminate the fractions before solving?
1/2x-5/4+2x=5/6+x

Answers

Answer:

12

Step-by-step explanation:

Find the common difference and the SIMPLIFIED general term formula.
3) a 14=1322 and a 38 = 3722

I reallyyyy need help!!! ASAP PLEASE!!!!!

Answers

Answer:

[tex]a_{n}[/tex] = 100n - 78

Step-by-step explanation:

the nth term of an arithmetic sequence is

[tex]a_{n}[/tex] = a₁ + (n - 1)d

where a₁ is the first term and d the common difference

given a₁₄ = 1322 and a₃₈ = 3722 , then

a₁ + 13d = 1322 → (1)

a₁ + 37d = 3722 → (2)

subtract (1) from (2) term by term to eliminate a₁

24d = 2400 ( divide both sides by 24 )

d = 100

substitute d = 100 into either of the 2 equations and solve for a₁

substituting into (1)

a₁ + 13(100) = 1322

a₁ + 1300 = 1322 ( subtract 1300 from both sides )

a₁ = 22

then nth term formula is

[tex]a_{n}[/tex] = 22 + 100(n - 1) = 22 + 100n - 100 = 100n - 78

Ving tools to form the correct answer on the graph.
Graph the line that represents this equation:
y + 2 =
(x+2)
Drawing Tools
Select
Point
Line
Click on a tool to begin drawing.
10
N
27

Answers

The graph of the equation is shown in the attached picture which is a straight line.

What is a straight line?

A straight line is a combination of endless points joined on both sides of the point.

The slope 'm' of any straight line is given by:

[tex]\rm m =\dfrac{y_2-y_1}{x_2-x_1}[/tex]

We have a graph of a equation:

y + 2 = (x+2)

or

y = x

The above equation is a linear equation or straight line equation the slope of the equation is 1.

m = 1

The y-intercept of the equation is 0.

We can plot the equation y = x by using the slope and y-intercept.

Thus, the graph of the equation is shown in the attached picture which is a straight line.

Learn more about the straight line here:

brainly.com/question/3493733

#SPJ1

Other Questions
The term "Diaspora" refers to __________.A.the break of Christianity with JudaismB.the blending of Jewish and Muslim traditionsC.the spread of Islam to southern Europe and western AsiaD.the spread of Judaism through the Middle East and southern EuropePlease select the best answer from the choices provided. Given the points X(-2, 5) and Y(2, -3), find the coordinates of the point P ondirected line segment XY thatpartitions segment XY such that the ratio of XPto PY is 4:1(,3)5( 71/17, 173737)(7) What is Mg ^-2 and SO4^-2= find the new prices after the given percentage increases.a] 35 increased by 10%b} 105.99 increased by 12% A farmer has only goats and hens in his farm,on a particular day,he sold 12 animals (goats and hens) and the total number of their legs was 32. how many goats and how many hens did he sell that day? The discovery of Cro-Magnons buried with daggers, beads, and other status symbols provides evidence of a a. the belief in life after death b b. cooperative big-game hunting c c. their technological inferiority compared to the Neanderthals. d d. the existence of a spoken language. Catalina and Morgan are finding the length of the third side of the right triangle. Who is correct? Explain your reasoning. PLS HURRY What countries were allied in World War I and opposed in World War II? Russia and Germany Germany and Britain France and Britain America and Italy which type of review is more rigorous than a traditional literature review but less rigorous than the systematic review because it includes only published reports? need answer Asap! Lots of points! Match each law with the effect it had on the British population The diagonal of a TV is 30 inches long. Assuming that this diagonal forms a pair of 30-60-90 right triangles, what are the exact length and width of the TV? A. 60 inches by 60,3 inches B. 602 inches by 60 2 inches O C. 15/2 inches by 15/2 inches OD. 15 inches by 153 inches The length of an arc of a circle is 7.34 units, and the measure of the corresponding central angle is 81. What is the approximate length of the radius of the circle? Which rule can be used to describe the x-coordinates in the translation below ? two ships leave a port sailing 18km/h and 26 km/h. Their angle between their directions of travel from the port is 39 . How far part are the skips to the nearest km after 2 hours If b = 5, then the exact value of a is? Which of the following abiotic factors might have an effect on how much photosynthesis can occur by the producers in an ecosystem?The temperature in the environment.The temperature in the environment.The water quality in the environment.The water quality in the environment.The air quality in the environment.The air quality in the environment.All of the above can affect photosynthesis. The table shows the height of water in a pool as it is being filled. A table showing Height of Water in a Pool with two columns and six rows. The first column, Time in minutes, has the entries, 2, 4, 6, 8, 10. The second column, Height in inches, has the entries, 8, 12, 16, 20, 24. The slope of the line through the points is 2. Which statement describes how the slope relates to the height of the water in the pool? The height of the water increases 2 inches per minute. The height of the water decreases 2 inches per minute. The height of the water was 2 inches before any water was added. The height of the water will be 2 inches when the pool is filled. help me THE^Solve: x=4+(4x-4)OOOx = 2x = 10x = 2 or x = 10Ono real solutionDONE A bakery makes and sells designer cakes. The function d (x) = 125 representsthe income, in dollars, from selling a cakes. The function p (y) = y - 1620represents the monthly profit, in dollars, if the bakery makes y dollars in income forthe month. What is the profit from selling 30 designer cakes in a single month?